Let $sum_{k=1}^infty a_n$ be convergent show that $sum_{k=1}^infty n(a_n-a_{n+1})$ converges












5












$begingroup$


Let $sumlimits_{k=1}^infty a_n$ be a convergent series where $a_ngeq0$ and $(a_n)$ is a monotone decreasing sequence prove that the series $sumlimits_{k=1}^infty n(a_n-a_{n+1})$ also converges.



What I tried :



Let $(A_n)$ be the sequence of partial sums of the series $sumlimits_{k=1}^infty a_n$ and $(B_n)$ be the sequence of partial sums of the series $sumlimits_{k=1}^infty n(a_n-a_{n+1})$.



Since $A_n=sumlimits_{k=1}^n a_k$ and $B_n=sumlimits_{k=1}^n k(a_k-a_{k+1})$ we get that:



begin{align}
B_n&=A_n-na_{n+1}\
&=(a_1-a_{n+1})+(a_2-a_{n+1})+...+(a_n-a_{n+1})\
&>(a_1-a_n)+(a_2-a_n)+...+(a_n-a_n)\
&=B_{n-1}
end{align}



we see that $(B_n)$ is a monotone increasing sequence $...(1)$



and



$B_n=A_n-na_{n+1}<A_n$ this implies that the sequence $(B_n)$ is bounded above ...(2)



Therefore (from (1) and (2)) the sequence $(B_n)$ converges so the series $sumlimits_{k=1}^infty n(a_n-a_{n+1})$ also converges



Is my proof correct?










share|cite|improve this question











$endgroup$












  • $begingroup$
    Looks ok. Some simplifications: you know that $B_n - B_{n-1} = n(a_n - a_{n+1})$ which is $>0$. Another way to look at it: you know that $A_n$ converges and $B_n = A_n - na_{n+1}$. This means that the converge of $B_n$ is equivalent to showing $na_{n+1} to 0$ as $ntoinfty$.
    $endgroup$
    – Winther
    Dec 25 '18 at 12:48












  • $begingroup$
    Thank you , and from the second one do we get that the sum of $sum_{k=1}^infty n(a_n-a_{n+1})$ equals the sum of $sum_{k=1}^infty a_n$?
    $endgroup$
    – Maths Survivor
    Dec 25 '18 at 13:00








  • 1




    $begingroup$
    Taking the limit $ntoinfty$ of $B_n = A_n - na_{n+1}$. However disregard the last thing I said. The usual way of showing that $na_{n+1} to 0$ is to show that $sum n(a_n-a_{n+1})$ converges so that would be circular (see e.g. math.stackexchange.com/questions/383769/…). Your approach is good. btw that $B_n = A_n - na_{n+1}$ is a special case of summation by parts.
    $endgroup$
    – Winther
    Dec 25 '18 at 13:02












  • $begingroup$
    I can use another method to prove that $na_nto0$ when $n to infty$ without using the convergence of the series $sum_{k=1}^infty n(a_n-a_{n+1})$ , so from $B_n=A_n-na_n$ and since the sequences $(A_n)$ and $(na_n)$ converge when $n to infty $ implies that also the sequnce $(B_n)$ converges right?
    $endgroup$
    – Maths Survivor
    Dec 25 '18 at 13:18










  • $begingroup$
    Yes that's right
    $endgroup$
    – Winther
    Dec 25 '18 at 13:19
















5












$begingroup$


Let $sumlimits_{k=1}^infty a_n$ be a convergent series where $a_ngeq0$ and $(a_n)$ is a monotone decreasing sequence prove that the series $sumlimits_{k=1}^infty n(a_n-a_{n+1})$ also converges.



What I tried :



Let $(A_n)$ be the sequence of partial sums of the series $sumlimits_{k=1}^infty a_n$ and $(B_n)$ be the sequence of partial sums of the series $sumlimits_{k=1}^infty n(a_n-a_{n+1})$.



Since $A_n=sumlimits_{k=1}^n a_k$ and $B_n=sumlimits_{k=1}^n k(a_k-a_{k+1})$ we get that:



begin{align}
B_n&=A_n-na_{n+1}\
&=(a_1-a_{n+1})+(a_2-a_{n+1})+...+(a_n-a_{n+1})\
&>(a_1-a_n)+(a_2-a_n)+...+(a_n-a_n)\
&=B_{n-1}
end{align}



we see that $(B_n)$ is a monotone increasing sequence $...(1)$



and



$B_n=A_n-na_{n+1}<A_n$ this implies that the sequence $(B_n)$ is bounded above ...(2)



Therefore (from (1) and (2)) the sequence $(B_n)$ converges so the series $sumlimits_{k=1}^infty n(a_n-a_{n+1})$ also converges



Is my proof correct?










share|cite|improve this question











$endgroup$












  • $begingroup$
    Looks ok. Some simplifications: you know that $B_n - B_{n-1} = n(a_n - a_{n+1})$ which is $>0$. Another way to look at it: you know that $A_n$ converges and $B_n = A_n - na_{n+1}$. This means that the converge of $B_n$ is equivalent to showing $na_{n+1} to 0$ as $ntoinfty$.
    $endgroup$
    – Winther
    Dec 25 '18 at 12:48












  • $begingroup$
    Thank you , and from the second one do we get that the sum of $sum_{k=1}^infty n(a_n-a_{n+1})$ equals the sum of $sum_{k=1}^infty a_n$?
    $endgroup$
    – Maths Survivor
    Dec 25 '18 at 13:00








  • 1




    $begingroup$
    Taking the limit $ntoinfty$ of $B_n = A_n - na_{n+1}$. However disregard the last thing I said. The usual way of showing that $na_{n+1} to 0$ is to show that $sum n(a_n-a_{n+1})$ converges so that would be circular (see e.g. math.stackexchange.com/questions/383769/…). Your approach is good. btw that $B_n = A_n - na_{n+1}$ is a special case of summation by parts.
    $endgroup$
    – Winther
    Dec 25 '18 at 13:02












  • $begingroup$
    I can use another method to prove that $na_nto0$ when $n to infty$ without using the convergence of the series $sum_{k=1}^infty n(a_n-a_{n+1})$ , so from $B_n=A_n-na_n$ and since the sequences $(A_n)$ and $(na_n)$ converge when $n to infty $ implies that also the sequnce $(B_n)$ converges right?
    $endgroup$
    – Maths Survivor
    Dec 25 '18 at 13:18










  • $begingroup$
    Yes that's right
    $endgroup$
    – Winther
    Dec 25 '18 at 13:19














5












5








5


2



$begingroup$


Let $sumlimits_{k=1}^infty a_n$ be a convergent series where $a_ngeq0$ and $(a_n)$ is a monotone decreasing sequence prove that the series $sumlimits_{k=1}^infty n(a_n-a_{n+1})$ also converges.



What I tried :



Let $(A_n)$ be the sequence of partial sums of the series $sumlimits_{k=1}^infty a_n$ and $(B_n)$ be the sequence of partial sums of the series $sumlimits_{k=1}^infty n(a_n-a_{n+1})$.



Since $A_n=sumlimits_{k=1}^n a_k$ and $B_n=sumlimits_{k=1}^n k(a_k-a_{k+1})$ we get that:



begin{align}
B_n&=A_n-na_{n+1}\
&=(a_1-a_{n+1})+(a_2-a_{n+1})+...+(a_n-a_{n+1})\
&>(a_1-a_n)+(a_2-a_n)+...+(a_n-a_n)\
&=B_{n-1}
end{align}



we see that $(B_n)$ is a monotone increasing sequence $...(1)$



and



$B_n=A_n-na_{n+1}<A_n$ this implies that the sequence $(B_n)$ is bounded above ...(2)



Therefore (from (1) and (2)) the sequence $(B_n)$ converges so the series $sumlimits_{k=1}^infty n(a_n-a_{n+1})$ also converges



Is my proof correct?










share|cite|improve this question











$endgroup$




Let $sumlimits_{k=1}^infty a_n$ be a convergent series where $a_ngeq0$ and $(a_n)$ is a monotone decreasing sequence prove that the series $sumlimits_{k=1}^infty n(a_n-a_{n+1})$ also converges.



What I tried :



Let $(A_n)$ be the sequence of partial sums of the series $sumlimits_{k=1}^infty a_n$ and $(B_n)$ be the sequence of partial sums of the series $sumlimits_{k=1}^infty n(a_n-a_{n+1})$.



Since $A_n=sumlimits_{k=1}^n a_k$ and $B_n=sumlimits_{k=1}^n k(a_k-a_{k+1})$ we get that:



begin{align}
B_n&=A_n-na_{n+1}\
&=(a_1-a_{n+1})+(a_2-a_{n+1})+...+(a_n-a_{n+1})\
&>(a_1-a_n)+(a_2-a_n)+...+(a_n-a_n)\
&=B_{n-1}
end{align}



we see that $(B_n)$ is a monotone increasing sequence $...(1)$



and



$B_n=A_n-na_{n+1}<A_n$ this implies that the sequence $(B_n)$ is bounded above ...(2)



Therefore (from (1) and (2)) the sequence $(B_n)$ converges so the series $sumlimits_{k=1}^infty n(a_n-a_{n+1})$ also converges



Is my proof correct?







real-analysis sequences-and-series proof-verification convergence






share|cite|improve this question















share|cite|improve this question













share|cite|improve this question




share|cite|improve this question








edited Dec 25 '18 at 17:12









Henning Makholm

241k17308549




241k17308549










asked Dec 25 '18 at 12:34









Maths SurvivorMaths Survivor

502219




502219












  • $begingroup$
    Looks ok. Some simplifications: you know that $B_n - B_{n-1} = n(a_n - a_{n+1})$ which is $>0$. Another way to look at it: you know that $A_n$ converges and $B_n = A_n - na_{n+1}$. This means that the converge of $B_n$ is equivalent to showing $na_{n+1} to 0$ as $ntoinfty$.
    $endgroup$
    – Winther
    Dec 25 '18 at 12:48












  • $begingroup$
    Thank you , and from the second one do we get that the sum of $sum_{k=1}^infty n(a_n-a_{n+1})$ equals the sum of $sum_{k=1}^infty a_n$?
    $endgroup$
    – Maths Survivor
    Dec 25 '18 at 13:00








  • 1




    $begingroup$
    Taking the limit $ntoinfty$ of $B_n = A_n - na_{n+1}$. However disregard the last thing I said. The usual way of showing that $na_{n+1} to 0$ is to show that $sum n(a_n-a_{n+1})$ converges so that would be circular (see e.g. math.stackexchange.com/questions/383769/…). Your approach is good. btw that $B_n = A_n - na_{n+1}$ is a special case of summation by parts.
    $endgroup$
    – Winther
    Dec 25 '18 at 13:02












  • $begingroup$
    I can use another method to prove that $na_nto0$ when $n to infty$ without using the convergence of the series $sum_{k=1}^infty n(a_n-a_{n+1})$ , so from $B_n=A_n-na_n$ and since the sequences $(A_n)$ and $(na_n)$ converge when $n to infty $ implies that also the sequnce $(B_n)$ converges right?
    $endgroup$
    – Maths Survivor
    Dec 25 '18 at 13:18










  • $begingroup$
    Yes that's right
    $endgroup$
    – Winther
    Dec 25 '18 at 13:19


















  • $begingroup$
    Looks ok. Some simplifications: you know that $B_n - B_{n-1} = n(a_n - a_{n+1})$ which is $>0$. Another way to look at it: you know that $A_n$ converges and $B_n = A_n - na_{n+1}$. This means that the converge of $B_n$ is equivalent to showing $na_{n+1} to 0$ as $ntoinfty$.
    $endgroup$
    – Winther
    Dec 25 '18 at 12:48












  • $begingroup$
    Thank you , and from the second one do we get that the sum of $sum_{k=1}^infty n(a_n-a_{n+1})$ equals the sum of $sum_{k=1}^infty a_n$?
    $endgroup$
    – Maths Survivor
    Dec 25 '18 at 13:00








  • 1




    $begingroup$
    Taking the limit $ntoinfty$ of $B_n = A_n - na_{n+1}$. However disregard the last thing I said. The usual way of showing that $na_{n+1} to 0$ is to show that $sum n(a_n-a_{n+1})$ converges so that would be circular (see e.g. math.stackexchange.com/questions/383769/…). Your approach is good. btw that $B_n = A_n - na_{n+1}$ is a special case of summation by parts.
    $endgroup$
    – Winther
    Dec 25 '18 at 13:02












  • $begingroup$
    I can use another method to prove that $na_nto0$ when $n to infty$ without using the convergence of the series $sum_{k=1}^infty n(a_n-a_{n+1})$ , so from $B_n=A_n-na_n$ and since the sequences $(A_n)$ and $(na_n)$ converge when $n to infty $ implies that also the sequnce $(B_n)$ converges right?
    $endgroup$
    – Maths Survivor
    Dec 25 '18 at 13:18










  • $begingroup$
    Yes that's right
    $endgroup$
    – Winther
    Dec 25 '18 at 13:19
















$begingroup$
Looks ok. Some simplifications: you know that $B_n - B_{n-1} = n(a_n - a_{n+1})$ which is $>0$. Another way to look at it: you know that $A_n$ converges and $B_n = A_n - na_{n+1}$. This means that the converge of $B_n$ is equivalent to showing $na_{n+1} to 0$ as $ntoinfty$.
$endgroup$
– Winther
Dec 25 '18 at 12:48






$begingroup$
Looks ok. Some simplifications: you know that $B_n - B_{n-1} = n(a_n - a_{n+1})$ which is $>0$. Another way to look at it: you know that $A_n$ converges and $B_n = A_n - na_{n+1}$. This means that the converge of $B_n$ is equivalent to showing $na_{n+1} to 0$ as $ntoinfty$.
$endgroup$
– Winther
Dec 25 '18 at 12:48














$begingroup$
Thank you , and from the second one do we get that the sum of $sum_{k=1}^infty n(a_n-a_{n+1})$ equals the sum of $sum_{k=1}^infty a_n$?
$endgroup$
– Maths Survivor
Dec 25 '18 at 13:00






$begingroup$
Thank you , and from the second one do we get that the sum of $sum_{k=1}^infty n(a_n-a_{n+1})$ equals the sum of $sum_{k=1}^infty a_n$?
$endgroup$
– Maths Survivor
Dec 25 '18 at 13:00






1




1




$begingroup$
Taking the limit $ntoinfty$ of $B_n = A_n - na_{n+1}$. However disregard the last thing I said. The usual way of showing that $na_{n+1} to 0$ is to show that $sum n(a_n-a_{n+1})$ converges so that would be circular (see e.g. math.stackexchange.com/questions/383769/…). Your approach is good. btw that $B_n = A_n - na_{n+1}$ is a special case of summation by parts.
$endgroup$
– Winther
Dec 25 '18 at 13:02






$begingroup$
Taking the limit $ntoinfty$ of $B_n = A_n - na_{n+1}$. However disregard the last thing I said. The usual way of showing that $na_{n+1} to 0$ is to show that $sum n(a_n-a_{n+1})$ converges so that would be circular (see e.g. math.stackexchange.com/questions/383769/…). Your approach is good. btw that $B_n = A_n - na_{n+1}$ is a special case of summation by parts.
$endgroup$
– Winther
Dec 25 '18 at 13:02














$begingroup$
I can use another method to prove that $na_nto0$ when $n to infty$ without using the convergence of the series $sum_{k=1}^infty n(a_n-a_{n+1})$ , so from $B_n=A_n-na_n$ and since the sequences $(A_n)$ and $(na_n)$ converge when $n to infty $ implies that also the sequnce $(B_n)$ converges right?
$endgroup$
– Maths Survivor
Dec 25 '18 at 13:18




$begingroup$
I can use another method to prove that $na_nto0$ when $n to infty$ without using the convergence of the series $sum_{k=1}^infty n(a_n-a_{n+1})$ , so from $B_n=A_n-na_n$ and since the sequences $(A_n)$ and $(na_n)$ converge when $n to infty $ implies that also the sequnce $(B_n)$ converges right?
$endgroup$
– Maths Survivor
Dec 25 '18 at 13:18












$begingroup$
Yes that's right
$endgroup$
– Winther
Dec 25 '18 at 13:19




$begingroup$
Yes that's right
$endgroup$
– Winther
Dec 25 '18 at 13:19










2 Answers
2






active

oldest

votes


















2












$begingroup$

Some comments:




  1. The identity $B_n = A_n - n a_{n+1}$ is not quite obvious enough to state without proof. A quick induction proof would work, as would writing out the sums using ellipses (i.e. the symbol "$ldots$") and simplifying.


  2. Showing $B_n < A_n$ establishes an upper bound based on $n$, which is not allowed (e.g. $B_n le B_n$ always!). As $A_n$ (being the partial sums of a convergent series) is convergent, you can easily establish an upper bound (especially when you consider the fact that $A_n$ is increasing).


  3. You can more quickly establish that $B_n$ is increasing by observing that it is the sum of positive numbers.


  4. You could also further note that, if $lim B_n < lim A_n$, then $a_n$ is approximately a multiple of the harmonic series, which is divergent, thus the two series share the same sum. (This is not a criticism, just something worth noting).







share|cite|improve this answer









$endgroup$









  • 1




    $begingroup$
    "then $a_n$ is approximately a multiple of the harmonic series" what does this mean? Can you give more explanation to this sentence?
    $endgroup$
    – Maths Survivor
    Dec 25 '18 at 13:12






  • 1




    $begingroup$
    @MathsSurvivor If $A_n to A$ and $B_n to B$ with $A neq B$, then $$frac{a_{n+1}}{frac{1}{n}} = na_{n+1} = A_n - B_n to A - B neq 0,$$ so by the limit comparison test, $a_{n+1}$, when summed, is a divergent sequence. This contradicts $A_n$ converging.
    $endgroup$
    – Theo Bendit
    Dec 25 '18 at 14:02





















2












$begingroup$

Note that we can use $n=sum_{k=1}^n (1)$ to write



$$begin{align}
sum_{n=1}^N n(a_{n+1}-a_n)&=sum_{n=1}^N sum_{k=1}^n(a_{n+1}-a_n)\\
&=sum_{k=1}^N sum_{n=k}^N (a_{n+1}-a_n)\\
&=sum_{k=1}^N (a_{N+1}-a_k)\\
&=Na_{N+1}-sum_{k=1}^N a_ktag1
end{align}$$



Inasmuch as $a_nge 0$ monotonically decreases to $0$, and $sum_{k=1}^n a_n<infty$, we have $lim_{ntoinfty }na_n=0$. Hence, using $(1)$, we see that



$$begin{align}
lim_{Nto infty }sum_{n=1}^N n(a_{n+1}-a_n)&=lim_{Ntoinfty}left(Na_{N+1}-sum_{k=1}^N a_kright)\\
&=-sum_{n=1}^infty a_n
end{align}$$



from which we conclude that $sum_{n=1}^infty n(a_{n+1}-a_n)$ converges.






share|cite|improve this answer









$endgroup$













  • $begingroup$
    Winther's comment on the question suggests that showing $lim n a_n = 0$ is often done by first establishing $sum n(a_n - a_{n+1}) = sum a_n$. Out of curiosity, how would you establish $lim n a_n = 0$?
    $endgroup$
    – Theo Bendit
    Dec 25 '18 at 22:44












  • $begingroup$
    Note that $$(2n)a_{2n}le 2sum_{n+1}^{2n}a_kto 0$$since $a_nge0$ is monotonic and $sum_n a_n<infty.$.
    $endgroup$
    – Mark Viola
    Dec 26 '18 at 4:05













Your Answer





StackExchange.ifUsing("editor", function () {
return StackExchange.using("mathjaxEditing", function () {
StackExchange.MarkdownEditor.creationCallbacks.add(function (editor, postfix) {
StackExchange.mathjaxEditing.prepareWmdForMathJax(editor, postfix, [["$", "$"], ["\\(","\\)"]]);
});
});
}, "mathjax-editing");

StackExchange.ready(function() {
var channelOptions = {
tags: "".split(" "),
id: "69"
};
initTagRenderer("".split(" "), "".split(" "), channelOptions);

StackExchange.using("externalEditor", function() {
// Have to fire editor after snippets, if snippets enabled
if (StackExchange.settings.snippets.snippetsEnabled) {
StackExchange.using("snippets", function() {
createEditor();
});
}
else {
createEditor();
}
});

function createEditor() {
StackExchange.prepareEditor({
heartbeatType: 'answer',
autoActivateHeartbeat: false,
convertImagesToLinks: true,
noModals: true,
showLowRepImageUploadWarning: true,
reputationToPostImages: 10,
bindNavPrevention: true,
postfix: "",
imageUploader: {
brandingHtml: "Powered by u003ca class="icon-imgur-white" href="https://imgur.com/"u003eu003c/au003e",
contentPolicyHtml: "User contributions licensed under u003ca href="https://creativecommons.org/licenses/by-sa/3.0/"u003ecc by-sa 3.0 with attribution requiredu003c/au003e u003ca href="https://stackoverflow.com/legal/content-policy"u003e(content policy)u003c/au003e",
allowUrls: true
},
noCode: true, onDemand: true,
discardSelector: ".discard-answer"
,immediatelyShowMarkdownHelp:true
});


}
});














draft saved

draft discarded


















StackExchange.ready(
function () {
StackExchange.openid.initPostLogin('.new-post-login', 'https%3a%2f%2fmath.stackexchange.com%2fquestions%2f3052073%2flet-sum-k-1-infty-a-n-be-convergent-show-that-sum-k-1-infty-na-n-a%23new-answer', 'question_page');
}
);

Post as a guest















Required, but never shown

























2 Answers
2






active

oldest

votes








2 Answers
2






active

oldest

votes









active

oldest

votes






active

oldest

votes









2












$begingroup$

Some comments:




  1. The identity $B_n = A_n - n a_{n+1}$ is not quite obvious enough to state without proof. A quick induction proof would work, as would writing out the sums using ellipses (i.e. the symbol "$ldots$") and simplifying.


  2. Showing $B_n < A_n$ establishes an upper bound based on $n$, which is not allowed (e.g. $B_n le B_n$ always!). As $A_n$ (being the partial sums of a convergent series) is convergent, you can easily establish an upper bound (especially when you consider the fact that $A_n$ is increasing).


  3. You can more quickly establish that $B_n$ is increasing by observing that it is the sum of positive numbers.


  4. You could also further note that, if $lim B_n < lim A_n$, then $a_n$ is approximately a multiple of the harmonic series, which is divergent, thus the two series share the same sum. (This is not a criticism, just something worth noting).







share|cite|improve this answer









$endgroup$









  • 1




    $begingroup$
    "then $a_n$ is approximately a multiple of the harmonic series" what does this mean? Can you give more explanation to this sentence?
    $endgroup$
    – Maths Survivor
    Dec 25 '18 at 13:12






  • 1




    $begingroup$
    @MathsSurvivor If $A_n to A$ and $B_n to B$ with $A neq B$, then $$frac{a_{n+1}}{frac{1}{n}} = na_{n+1} = A_n - B_n to A - B neq 0,$$ so by the limit comparison test, $a_{n+1}$, when summed, is a divergent sequence. This contradicts $A_n$ converging.
    $endgroup$
    – Theo Bendit
    Dec 25 '18 at 14:02


















2












$begingroup$

Some comments:




  1. The identity $B_n = A_n - n a_{n+1}$ is not quite obvious enough to state without proof. A quick induction proof would work, as would writing out the sums using ellipses (i.e. the symbol "$ldots$") and simplifying.


  2. Showing $B_n < A_n$ establishes an upper bound based on $n$, which is not allowed (e.g. $B_n le B_n$ always!). As $A_n$ (being the partial sums of a convergent series) is convergent, you can easily establish an upper bound (especially when you consider the fact that $A_n$ is increasing).


  3. You can more quickly establish that $B_n$ is increasing by observing that it is the sum of positive numbers.


  4. You could also further note that, if $lim B_n < lim A_n$, then $a_n$ is approximately a multiple of the harmonic series, which is divergent, thus the two series share the same sum. (This is not a criticism, just something worth noting).







share|cite|improve this answer









$endgroup$









  • 1




    $begingroup$
    "then $a_n$ is approximately a multiple of the harmonic series" what does this mean? Can you give more explanation to this sentence?
    $endgroup$
    – Maths Survivor
    Dec 25 '18 at 13:12






  • 1




    $begingroup$
    @MathsSurvivor If $A_n to A$ and $B_n to B$ with $A neq B$, then $$frac{a_{n+1}}{frac{1}{n}} = na_{n+1} = A_n - B_n to A - B neq 0,$$ so by the limit comparison test, $a_{n+1}$, when summed, is a divergent sequence. This contradicts $A_n$ converging.
    $endgroup$
    – Theo Bendit
    Dec 25 '18 at 14:02
















2












2








2





$begingroup$

Some comments:




  1. The identity $B_n = A_n - n a_{n+1}$ is not quite obvious enough to state without proof. A quick induction proof would work, as would writing out the sums using ellipses (i.e. the symbol "$ldots$") and simplifying.


  2. Showing $B_n < A_n$ establishes an upper bound based on $n$, which is not allowed (e.g. $B_n le B_n$ always!). As $A_n$ (being the partial sums of a convergent series) is convergent, you can easily establish an upper bound (especially when you consider the fact that $A_n$ is increasing).


  3. You can more quickly establish that $B_n$ is increasing by observing that it is the sum of positive numbers.


  4. You could also further note that, if $lim B_n < lim A_n$, then $a_n$ is approximately a multiple of the harmonic series, which is divergent, thus the two series share the same sum. (This is not a criticism, just something worth noting).







share|cite|improve this answer









$endgroup$



Some comments:




  1. The identity $B_n = A_n - n a_{n+1}$ is not quite obvious enough to state without proof. A quick induction proof would work, as would writing out the sums using ellipses (i.e. the symbol "$ldots$") and simplifying.


  2. Showing $B_n < A_n$ establishes an upper bound based on $n$, which is not allowed (e.g. $B_n le B_n$ always!). As $A_n$ (being the partial sums of a convergent series) is convergent, you can easily establish an upper bound (especially when you consider the fact that $A_n$ is increasing).


  3. You can more quickly establish that $B_n$ is increasing by observing that it is the sum of positive numbers.


  4. You could also further note that, if $lim B_n < lim A_n$, then $a_n$ is approximately a multiple of the harmonic series, which is divergent, thus the two series share the same sum. (This is not a criticism, just something worth noting).








share|cite|improve this answer












share|cite|improve this answer



share|cite|improve this answer










answered Dec 25 '18 at 13:05









Theo BenditTheo Bendit

19.4k12353




19.4k12353








  • 1




    $begingroup$
    "then $a_n$ is approximately a multiple of the harmonic series" what does this mean? Can you give more explanation to this sentence?
    $endgroup$
    – Maths Survivor
    Dec 25 '18 at 13:12






  • 1




    $begingroup$
    @MathsSurvivor If $A_n to A$ and $B_n to B$ with $A neq B$, then $$frac{a_{n+1}}{frac{1}{n}} = na_{n+1} = A_n - B_n to A - B neq 0,$$ so by the limit comparison test, $a_{n+1}$, when summed, is a divergent sequence. This contradicts $A_n$ converging.
    $endgroup$
    – Theo Bendit
    Dec 25 '18 at 14:02
















  • 1




    $begingroup$
    "then $a_n$ is approximately a multiple of the harmonic series" what does this mean? Can you give more explanation to this sentence?
    $endgroup$
    – Maths Survivor
    Dec 25 '18 at 13:12






  • 1




    $begingroup$
    @MathsSurvivor If $A_n to A$ and $B_n to B$ with $A neq B$, then $$frac{a_{n+1}}{frac{1}{n}} = na_{n+1} = A_n - B_n to A - B neq 0,$$ so by the limit comparison test, $a_{n+1}$, when summed, is a divergent sequence. This contradicts $A_n$ converging.
    $endgroup$
    – Theo Bendit
    Dec 25 '18 at 14:02










1




1




$begingroup$
"then $a_n$ is approximately a multiple of the harmonic series" what does this mean? Can you give more explanation to this sentence?
$endgroup$
– Maths Survivor
Dec 25 '18 at 13:12




$begingroup$
"then $a_n$ is approximately a multiple of the harmonic series" what does this mean? Can you give more explanation to this sentence?
$endgroup$
– Maths Survivor
Dec 25 '18 at 13:12




1




1




$begingroup$
@MathsSurvivor If $A_n to A$ and $B_n to B$ with $A neq B$, then $$frac{a_{n+1}}{frac{1}{n}} = na_{n+1} = A_n - B_n to A - B neq 0,$$ so by the limit comparison test, $a_{n+1}$, when summed, is a divergent sequence. This contradicts $A_n$ converging.
$endgroup$
– Theo Bendit
Dec 25 '18 at 14:02






$begingroup$
@MathsSurvivor If $A_n to A$ and $B_n to B$ with $A neq B$, then $$frac{a_{n+1}}{frac{1}{n}} = na_{n+1} = A_n - B_n to A - B neq 0,$$ so by the limit comparison test, $a_{n+1}$, when summed, is a divergent sequence. This contradicts $A_n$ converging.
$endgroup$
– Theo Bendit
Dec 25 '18 at 14:02













2












$begingroup$

Note that we can use $n=sum_{k=1}^n (1)$ to write



$$begin{align}
sum_{n=1}^N n(a_{n+1}-a_n)&=sum_{n=1}^N sum_{k=1}^n(a_{n+1}-a_n)\\
&=sum_{k=1}^N sum_{n=k}^N (a_{n+1}-a_n)\\
&=sum_{k=1}^N (a_{N+1}-a_k)\\
&=Na_{N+1}-sum_{k=1}^N a_ktag1
end{align}$$



Inasmuch as $a_nge 0$ monotonically decreases to $0$, and $sum_{k=1}^n a_n<infty$, we have $lim_{ntoinfty }na_n=0$. Hence, using $(1)$, we see that



$$begin{align}
lim_{Nto infty }sum_{n=1}^N n(a_{n+1}-a_n)&=lim_{Ntoinfty}left(Na_{N+1}-sum_{k=1}^N a_kright)\\
&=-sum_{n=1}^infty a_n
end{align}$$



from which we conclude that $sum_{n=1}^infty n(a_{n+1}-a_n)$ converges.






share|cite|improve this answer









$endgroup$













  • $begingroup$
    Winther's comment on the question suggests that showing $lim n a_n = 0$ is often done by first establishing $sum n(a_n - a_{n+1}) = sum a_n$. Out of curiosity, how would you establish $lim n a_n = 0$?
    $endgroup$
    – Theo Bendit
    Dec 25 '18 at 22:44












  • $begingroup$
    Note that $$(2n)a_{2n}le 2sum_{n+1}^{2n}a_kto 0$$since $a_nge0$ is monotonic and $sum_n a_n<infty.$.
    $endgroup$
    – Mark Viola
    Dec 26 '18 at 4:05


















2












$begingroup$

Note that we can use $n=sum_{k=1}^n (1)$ to write



$$begin{align}
sum_{n=1}^N n(a_{n+1}-a_n)&=sum_{n=1}^N sum_{k=1}^n(a_{n+1}-a_n)\\
&=sum_{k=1}^N sum_{n=k}^N (a_{n+1}-a_n)\\
&=sum_{k=1}^N (a_{N+1}-a_k)\\
&=Na_{N+1}-sum_{k=1}^N a_ktag1
end{align}$$



Inasmuch as $a_nge 0$ monotonically decreases to $0$, and $sum_{k=1}^n a_n<infty$, we have $lim_{ntoinfty }na_n=0$. Hence, using $(1)$, we see that



$$begin{align}
lim_{Nto infty }sum_{n=1}^N n(a_{n+1}-a_n)&=lim_{Ntoinfty}left(Na_{N+1}-sum_{k=1}^N a_kright)\\
&=-sum_{n=1}^infty a_n
end{align}$$



from which we conclude that $sum_{n=1}^infty n(a_{n+1}-a_n)$ converges.






share|cite|improve this answer









$endgroup$













  • $begingroup$
    Winther's comment on the question suggests that showing $lim n a_n = 0$ is often done by first establishing $sum n(a_n - a_{n+1}) = sum a_n$. Out of curiosity, how would you establish $lim n a_n = 0$?
    $endgroup$
    – Theo Bendit
    Dec 25 '18 at 22:44












  • $begingroup$
    Note that $$(2n)a_{2n}le 2sum_{n+1}^{2n}a_kto 0$$since $a_nge0$ is monotonic and $sum_n a_n<infty.$.
    $endgroup$
    – Mark Viola
    Dec 26 '18 at 4:05
















2












2








2





$begingroup$

Note that we can use $n=sum_{k=1}^n (1)$ to write



$$begin{align}
sum_{n=1}^N n(a_{n+1}-a_n)&=sum_{n=1}^N sum_{k=1}^n(a_{n+1}-a_n)\\
&=sum_{k=1}^N sum_{n=k}^N (a_{n+1}-a_n)\\
&=sum_{k=1}^N (a_{N+1}-a_k)\\
&=Na_{N+1}-sum_{k=1}^N a_ktag1
end{align}$$



Inasmuch as $a_nge 0$ monotonically decreases to $0$, and $sum_{k=1}^n a_n<infty$, we have $lim_{ntoinfty }na_n=0$. Hence, using $(1)$, we see that



$$begin{align}
lim_{Nto infty }sum_{n=1}^N n(a_{n+1}-a_n)&=lim_{Ntoinfty}left(Na_{N+1}-sum_{k=1}^N a_kright)\\
&=-sum_{n=1}^infty a_n
end{align}$$



from which we conclude that $sum_{n=1}^infty n(a_{n+1}-a_n)$ converges.






share|cite|improve this answer









$endgroup$



Note that we can use $n=sum_{k=1}^n (1)$ to write



$$begin{align}
sum_{n=1}^N n(a_{n+1}-a_n)&=sum_{n=1}^N sum_{k=1}^n(a_{n+1}-a_n)\\
&=sum_{k=1}^N sum_{n=k}^N (a_{n+1}-a_n)\\
&=sum_{k=1}^N (a_{N+1}-a_k)\\
&=Na_{N+1}-sum_{k=1}^N a_ktag1
end{align}$$



Inasmuch as $a_nge 0$ monotonically decreases to $0$, and $sum_{k=1}^n a_n<infty$, we have $lim_{ntoinfty }na_n=0$. Hence, using $(1)$, we see that



$$begin{align}
lim_{Nto infty }sum_{n=1}^N n(a_{n+1}-a_n)&=lim_{Ntoinfty}left(Na_{N+1}-sum_{k=1}^N a_kright)\\
&=-sum_{n=1}^infty a_n
end{align}$$



from which we conclude that $sum_{n=1}^infty n(a_{n+1}-a_n)$ converges.







share|cite|improve this answer












share|cite|improve this answer



share|cite|improve this answer










answered Dec 25 '18 at 17:11









Mark ViolaMark Viola

133k1277176




133k1277176












  • $begingroup$
    Winther's comment on the question suggests that showing $lim n a_n = 0$ is often done by first establishing $sum n(a_n - a_{n+1}) = sum a_n$. Out of curiosity, how would you establish $lim n a_n = 0$?
    $endgroup$
    – Theo Bendit
    Dec 25 '18 at 22:44












  • $begingroup$
    Note that $$(2n)a_{2n}le 2sum_{n+1}^{2n}a_kto 0$$since $a_nge0$ is monotonic and $sum_n a_n<infty.$.
    $endgroup$
    – Mark Viola
    Dec 26 '18 at 4:05




















  • $begingroup$
    Winther's comment on the question suggests that showing $lim n a_n = 0$ is often done by first establishing $sum n(a_n - a_{n+1}) = sum a_n$. Out of curiosity, how would you establish $lim n a_n = 0$?
    $endgroup$
    – Theo Bendit
    Dec 25 '18 at 22:44












  • $begingroup$
    Note that $$(2n)a_{2n}le 2sum_{n+1}^{2n}a_kto 0$$since $a_nge0$ is monotonic and $sum_n a_n<infty.$.
    $endgroup$
    – Mark Viola
    Dec 26 '18 at 4:05


















$begingroup$
Winther's comment on the question suggests that showing $lim n a_n = 0$ is often done by first establishing $sum n(a_n - a_{n+1}) = sum a_n$. Out of curiosity, how would you establish $lim n a_n = 0$?
$endgroup$
– Theo Bendit
Dec 25 '18 at 22:44






$begingroup$
Winther's comment on the question suggests that showing $lim n a_n = 0$ is often done by first establishing $sum n(a_n - a_{n+1}) = sum a_n$. Out of curiosity, how would you establish $lim n a_n = 0$?
$endgroup$
– Theo Bendit
Dec 25 '18 at 22:44














$begingroup$
Note that $$(2n)a_{2n}le 2sum_{n+1}^{2n}a_kto 0$$since $a_nge0$ is monotonic and $sum_n a_n<infty.$.
$endgroup$
– Mark Viola
Dec 26 '18 at 4:05






$begingroup$
Note that $$(2n)a_{2n}le 2sum_{n+1}^{2n}a_kto 0$$since $a_nge0$ is monotonic and $sum_n a_n<infty.$.
$endgroup$
– Mark Viola
Dec 26 '18 at 4:05




















draft saved

draft discarded




















































Thanks for contributing an answer to Mathematics Stack Exchange!


  • Please be sure to answer the question. Provide details and share your research!

But avoid



  • Asking for help, clarification, or responding to other answers.

  • Making statements based on opinion; back them up with references or personal experience.


Use MathJax to format equations. MathJax reference.


To learn more, see our tips on writing great answers.




draft saved


draft discarded














StackExchange.ready(
function () {
StackExchange.openid.initPostLogin('.new-post-login', 'https%3a%2f%2fmath.stackexchange.com%2fquestions%2f3052073%2flet-sum-k-1-infty-a-n-be-convergent-show-that-sum-k-1-infty-na-n-a%23new-answer', 'question_page');
}
);

Post as a guest















Required, but never shown





















































Required, but never shown














Required, but never shown












Required, but never shown







Required, but never shown

































Required, but never shown














Required, but never shown












Required, but never shown







Required, but never shown







Popular posts from this blog

To store a contact into the json file from server.js file using a class in NodeJS

Redirect URL with Chrome Remote Debugging Android Devices

Dieringhausen